LSAT and Law School Admissions Forum

Get expert LSAT preparation and law school admissions advice from PowerScore Test Preparation.

 Administrator
PowerScore Staff
  • PowerScore Staff
  • Posts: 8916
  • Joined: Feb 02, 2011
|
#101150
Complete Question Explanation

The correct answer choice is (B).

Answer choice (A):

Answer choice (B): This is the correct answer choice.

Answer choice (C):

Answer choice (D):

Answer choice (E):

This explanation is still in progress. Please post any questions below!
 Jay
  • Posts: 46
  • Joined: Jan 09, 2020
|
#76115
Could someone explain why the correct answer is (B) , but not (E)?
 Christen Hammock
PowerScore Staff
  • PowerScore Staff
  • Posts: 61
  • Joined: May 14, 2020
|
#76333
Hey Jay!

Answer Choice (E) doesn't fit the passage's description of the second rationale. There's nothing "inherent" in the level of punishments—one is based on social benefit, and the other is based on the severity of the crime. The difference is that, as answer choice (B) points out, the second rationale doesn't suffer from the problem of disproportionate punishments for harm. The shoplifting example around line 25 supports this: the first rationale might lead us to punish this relatively low-harm crime harshly so that shoplifting is deterred. That's the "potential unfairness" in (B).


Christen

Get the most out of your LSAT Prep Plus subscription.

Analyze and track your performance with our Testing and Analytics Package.